(p, q) = (1, 2), (1, 3), (2, 1), (2, 2), (2, 5), (3, 1), (3, 5), (5, 2), (5, 3).
Applying the restriction: p≤ q, we have:
(p,q) = (1.2), (1,3), (2,2), (2,3), (3,5)
EXPLANATION: Refer to the solution submitted by Brian Smith in this location. (p,q) = (1, 2), (1, 3), (2, 1), (2, 2), (2, 5), (3, 1), (3, 5), (5, 2), (5, 3)
In consonance with the Solution to Problem B1 of 35th IMO 1994, the possible solutions without the restriction p<=q are given by:
(p,q) = (1, 2), (1, 3), (2, 1), (2, 2), (2, 5), (3, 1), (3, 5), (5, 2), (5, 3).
|